$\begingroup$

Based off of this puzzle and this Youtube video

Rules:

  • You control the pieces labeled kn (knight), b (bishop), and r (rook)
  • You are trying to capture the piece labeled ok (opponent king)
  • You move pieces as usual, however after every move, unless there is a black square under them (labeled x in the text version) or it is on the floor or there is another piece that you control beneath it, the piece that you moved will fall until it hits one of the three things that I mentioned.
  • If a piece hits the ok, it is captured.
  • ok cannot move.
  • Walls and floors can be passed through by kn.

The puzzle:

+---+---+---+---+---+---+---+---+
|   |   |   | x |   |   |   |   |
+---+---+---+---+---+---+---+---+
|   | x | x | x |   |   |   | x |
+---+---+---+---+---+---+---+---+
|   | x |kn | x |   |   | x |   |
+---+---+---+---+---+---+---+---+
|   | x | x | x | x | x | x | x |
+---+---+---+---+---+---+---+---+
|   | x |   | x |   | x |   |ok |
+---+---+---+---+---+---+---+---+
|   |   | x |   |   | x |   | x |
+---+---+---+---+---+---+---+---+
| b | x |   |   | x |   |   |   |
+---+---+---+---+---+---+---+---+
| r |   |   | x |   |   |   |   |
+---+---+---+---+---+---+---+---+

enter image description here

Now my question to you is:

What is the minimum number of moves needed to capture the opponent’s king?

$\endgroup$

$\begingroup$

The minumum number of moves appears to be

Four

four moves

$\endgroup$

12